Limit Of $(1+ X/n)^n$ When $n$ Tends To Infinity - Math Stack Exchange
-
- Home
- Questions
- Tags
- Users
- Unanswered
- Teams
Ask questions, find answers and collaborate at work with Stack Overflow for Teams.
Try Teams for free Explore Teams - Teams
-
Ask questions, find answers and collaborate at work with Stack Overflow for Teams. Explore Teams
Teams
Q&A for work
Connect and share knowledge within a single location that is structured and easy to search.
Learn more about Teams Limit of $(1+ x/n)^n$ when $n$ tends to infinity [duplicate] Ask Question Asked 10 years, 4 months ago Modified 6 years, 7 months ago Viewed 380k times 49 $\begingroup$ This question already has answers here: About $\lim \left(1+\frac {x}{n}\right)^n$ (14 answers) Closed 10 years ago.Does anyone know the exact proof of this limit result?
$$\lim_{n\to\infty} \left(1+\frac{x}{n}\right)^n = e^x$$
Share Cite Follow edited Jul 30, 2014 at 15:32 Darth Geek 12.4k1 gold badge34 silver badges52 bronze badges asked Jul 30, 2014 at 15:30 narendra-choudharynarendra-choudhary 7881 gold badge8 silver badges14 bronze badges $\endgroup$ 6- 2 $\begingroup$ Depends on the definition of $e^x$... $\endgroup$ – Thomas Andrews Commented Jul 30, 2014 at 15:32
- $\begingroup$ have you tried to explicit the power and take the limit? It converge to the taylor series. $\endgroup$ – Lolman Commented Jul 30, 2014 at 15:33
- $\begingroup$ I tried by taking log of both sides, but I don't know what to do after this step. Thought of using L'Hopital's rule. But that ain't helping. $\endgroup$ – narendra-choudhary Commented Jul 30, 2014 at 15:38
- $\begingroup$ You can use this technique. $\endgroup$ – Mhenni Benghorbal Commented Jul 30, 2014 at 15:44
- 5 $\begingroup$ Using L'Hopital on the log you get: $$\lim_{n\to\infty} \frac{\log(1+\frac{x}{n})}{1/n}=\lim_{n\to\infty}\frac{n}{n+x}\frac{-x}{n^2}(- n^2)=x$$ $\endgroup$ – Lolman Commented Jul 30, 2014 at 15:46
3 Answers
Sorted by: Reset to default Highest score (default) Date modified (newest first) Date created (oldest first) 84 $\begingroup$A short proof:
$\left(1+\frac{x}{n}\right)^n = e^{n\log\left(1+\dfrac{x}{n}\right)}$
Since $\log(1+x) = x + O(x^2)$ when $x \to 0$, we have $n\log(1 + \frac{x}{n}) = x + O(\frac{x^2}{n})$ when $n\to +\infty$
Share Cite Follow edited Jul 30, 2014 at 15:37 Thomas Andrews 180k18 gold badges219 silver badges413 bronze badges answered Jul 30, 2014 at 15:36 Petite EtincellePetite Etincelle 14.9k2 gold badges33 silver badges61 bronze badges $\endgroup$ 7- 4 $\begingroup$ Or just use the derivate of $\log$ to evaluate $\lim_{y\to 0}\frac{\log(1+xy)}{y}$, then let $y=1/n$ to get the limit of $n\log(1+x/n)$... $\endgroup$ – Thomas Andrews Commented Jul 30, 2014 at 15:39
- 2 $\begingroup$ Even more elementary would be to use the basic inequality $\frac{x/n}{1+x/n}\le \log(1+x/n)\le x/n$. Then a straightforward application of the squeeze theorem does the trick. $\endgroup$ – Mark Viola Commented Oct 25, 2015 at 16:03
- 2 $\begingroup$ In this case, why does writing $(1+\frac{x}{n})^n=10^{n\log_{\, 10} \, \, (1+\frac{x}{n})}$ not show that $\lim_{n \to \infty}(1+\frac{x}{n})^n=10^x$? $\endgroup$ – Rasputin Commented Jan 19, 2019 at 21:17
- 6 $\begingroup$ @Rasputin because the Taylor expansion of $\log_{10} (1+x)$ is different $\endgroup$ – Petite Etincelle Commented Jan 20, 2019 at 13:27
- 1 $\begingroup$ What is $O(x^2)$? $\endgroup$ – PGupta Commented Sep 19, 2020 at 17:00
$$e^{\ln{(1 + \frac{x}{n})^n} }=e^{n \ln(1+\frac{x}{n})}$$
$$\lim_{n \to +\infty} (1 + \frac{x}{n})^n =\lim_{n \to +\infty} e^{n \ln(1+\frac{x}{n})} \\ =e^{\lim_{n \to +\infty} n \ln(1+\frac{x}{n})} =e^{\lim_{n \to +\infty}\frac{ \ln(1+\frac{x}{n})}{\frac{1}{n}}}$$
Apply L'Hopital's Rule:
$$=e^{\lim_{n \to +\infty}\frac{(\frac{-x}{n^2})\frac{1}{1+\frac{x}{n}}}{-\frac{1}{n^2}}} =e^{\lim_{n \to +\infty}\frac{x}{1+\frac{x}{n}}} =e^x$$
Therefore, $$(1+\frac{x}{n})^n \to e^x$$
Share Cite Follow edited May 2, 2018 at 20:27 CommunityBot 1 answered Jul 30, 2014 at 15:46 evindaevinda 7,9596 gold badges42 silver badges88 bronze badges $\endgroup$ 6- 2 $\begingroup$ First: Is the first result in the fourth row by L'Hopital's rule? Second, isn't a limit missing in the second result of the fourth row? $exp(x/(1+x/n))$ should be $exp\{lim[x/(1+x/n)]\}$ $\endgroup$ – Carol Eisen Commented Dec 25, 2017 at 17:39
- 1 $\begingroup$ You are correct! I edited it to account for the missing limit. Also, you are correct in that one of the steps is using L'Hopital's Rule. $\endgroup$ – H. Dewey Commented May 2, 2018 at 19:58
- $\begingroup$ Should I have permission to apply L' Hospital as it is n tends to infinity not x ,I mean it is kind of discrite case not continuous. $\endgroup$ – Supriyo Banerjee Commented Apr 24, 2019 at 14:53
- $\begingroup$ I understand everything except the part $\dfrac{-x}{n^{2}}$. Where does this come from? Isn't the derivative of $\ln(1+\frac{x}{n})=\frac{1}{1+\frac{x}{n}}$ only? $\endgroup$ – James Warthington Commented Oct 12, 2019 at 19:21
- 2 $\begingroup$ you're differentiating with respect to x, not n, my man. The variable in the limit is N not x. $\endgroup$ – astralwolf Commented Dec 23, 2019 at 19:45
You can use the binomial series expansion. For example:
$$\left(1+\frac{x}{n}\right)^n =1+ \frac{n}{1!}\left(\frac{x}{n}\right)^1+\frac{n(n-1)}{2!}\left(\frac{x}{n}\right)^2+\frac{n(n-1)(n-2)}{3!}\left(\frac{x}{n}\right)^3+\cdots $$
$$\left(1+\frac{x}{n}\right)^n =1+ \frac{n}{n}x+\frac{n(n-1)}{n^2}\frac{x^2}{2!}+\frac{n(n-1)(n-2)}{n^3}\frac{x^3}{3!} + \cdots$$
As $n \to \infty$ the coefficients in $n$ all tend to $1$. Hence:
$$\lim_{n \to \infty}\left(1+\frac{x}{n}\right)^n = 1+x+\frac{x^2}{2!}+\frac{x^3}{3!}+\cdots $$ You'll recognise this last power series as the Taylor series for $\mathrm{e}^x$.
Share Cite Follow edited Aug 20, 2016 at 19:11 answered Jul 30, 2014 at 15:39 Fly by NightFly by Night 32.7k4 gold badges56 silver badges103 bronze badges $\endgroup$ 5- 16 $\begingroup$ what if there are infinite terms and then you cannot exchange the sum and limit if n is non integer $\endgroup$ – happymath Commented Mar 3, 2015 at 10:17
- 3 $\begingroup$ it's not easy to make this argument rigorous. This answer would be better if it pointed out that fact $\endgroup$ – wlad Commented May 30, 2019 at 18:28
- 4 $\begingroup$ a rigorous version of this argument uses the monotone convergence theorem $\endgroup$ – wlad Commented May 30, 2019 at 18:41
- $\begingroup$ rigorous version: math.stackexchange.com/a/1898375/445105 $\endgroup$ – Felix Benning Commented Sep 17, 2021 at 10:03
- $\begingroup$ @happymath $n$ is Never Non Ninteger :-) $\endgroup$ – Oskar Limka Commented Oct 12, 2021 at 17:08
Not the answer you're looking for? Browse other questions tagged
.- Featured on Meta
- We’re (finally!) going to the cloud!
- More network sites to see advertising test [updated with phase 2]
Linked
55 About $\lim \left(1+\frac {x}{n}\right)^n$ 17 If $z_n \to z$ then $(1+z_n/n)^n \to e^z$ 9 Limit of $(1+1/n)^n$ is not equal to one, but why ? 18 How to calculate the limit $\lim_{n\to\infty}\left(\frac{1}{\sqrt{n^2+n+1}}+\frac{1}{\sqrt{n^2+n+2}}+\cdots+\frac{1}{\sqrt{n^2+n+n}}\right)^n$ 9 Geometric distribution converges to exponential distribution 2 Limit of $\left(1+\frac{1}{2n}\right)^n$ 0 Stuck in the proof of $(1+\frac{x}{n})^n \to e^x$ 2 Definition of e, and proof of existence. 3 Finding the convergence 2 A question regarding the prime-number theorem See more linked questionsRelated
0 A limit to infinity 1 The limit of product of two functions, where one tends to infinity and the other to zero 2 Limit of geometric series when common ratio tends to 1 9 limit when zero divided by infinity 0 Calculating limit for sequence that tends to infinity 0 What is the limit of series when n tends to infinity? 2 Find the limit of a multiplying term function when n tends to infinity. 3 L'Hospital when limit tends to infinity 0 Limit of the hypergeometric function with x tends infinityHot Network Questions
- Can you omit から directly connecting plain forms?
- Prospective employers tell me my field is obsolete. How can I reinvent myself?
- I have a Greek seasonal residence permit. Can I go back to my country from Poland instead of Greece without any issue?
- Can not update chrome in ubuntu
- Criticisms of "reductio ad absurdum" argumentation?
- The usage of Select for list (deep level)
- Is it (always) better to build a model prior to viewing the data?
- PSE Advent Calendar 2024 (Day 1): A Snowy Christmas
- How to calibrate fancy headers?
- How do I report to Springer a scientific fraud to a cryptographic paper published in Springer?
- In AES GCM, would using different nonces that are close reveal data?
- Calculating the wattage of a speaker protection light bulb
- To which extent I should let my PI know that I am not feeling very well with my PhD study
- Is it possible to use NAS hard drives in a desktop?
- Why should C++ uint8_t data not be printable?
- Can Trump reverse President Joe Biden's pardon for his son?
- Use of “12 m.” for noon and “12 p.m.” for midnight
- What are Christian responses to Carlo Alvaro's argument against Christian theism?
- Please explain the end of Tosefta Eduyot
- Why do you want a taut surface on dough?
- Example of non homogenous manifold with a finitely generated algebra of natural functions
- Is there just one Zero?
- How to set up a local VPN-connected environment with a public IP address?
- Number of roots of a quadratic form over GF2
Từ khóa » Xn+1-xn =a^n
-
ON THE RECURSIVE SEQUENCE Xn+1 = Xn-1/g(xn) - JSTOR
-
[PDF] Theorem - ( 1 + X N )n = Ex (= Exp(x)) - Lim
-
On The Recursive Sequence Xn+1 = α + Xn−1/xn
-
[PDF] Practice Problems 2: Convergence Of Sequences And Monotone ...
-
[PDF] Lecture 2 : Convergence Of A Sequence, Monotone Sequences
-
[PDF] Midterm Solutions
-
[PDF] Solutions To Homework 6- MAT319 - Stony Brook Math Department
-
[PDF] Sequences - UC Davis Math
-
[PDF] 447 HOMEWORK SET 6 1. 3.3 3) Let X1 ≥ 2 And X N+1 := 1 + √ Xn
-
On The Recursive Sequence Xn+1=−1/xn+A/xn−1 - ResearchGate
-
On The Recursive Sequence Xn+1 = Alpha+(xn-1/xn^k) - ResearchGate
-
[PDF] Section 12.9, Problem 38 - Math
-
[PDF] Solutions To Homework Set 3